Đến nội dung

yellow nội dung

Có 365 mục bởi yellow (Tìm giới hạn từ 06-06-2020)



Sắp theo                Sắp xếp  

#346934 CMR $DA, MP, CB$ đồng quy.

Đã gửi bởi yellow on 15-08-2012 - 15:26 trong Hình học

Cho hình thang $ABCD (AB //CD, AB < CD)$. Các đường phân giác đồng quy tại $I$. Đường thẳng qua $I$ vuông góc với $AB$ tại $P$; $CD$ tại $Q$. Trên $DC$ lấy điểm $M$ sao cho $CM = DQ$. CMR $DA, MP, CB$ đồng quy.



#359824 Phân thức

Đã gửi bởi yellow on 07-10-2012 - 18:17 trong Đại số

2,Giải phương trình
$\frac{(x-a)(x-c)}{(b-a)(b-c)}+\frac{(x-b)(x-c)}{(a-b)(a-c)}=1$

ĐKXĐ: $a, b, c$ khác nhau đôi một.
Ta có phương trình đã cho tương đương với:
$(x-c)\left [ \frac{a-x}{(a-b)(b-c)}+\frac{x-b}{(a-b)(a-c)} \right ]=1$
$\Leftrightarrow (x-c).\frac{(a-x)(a-c+(x-b)(b-c)}{(a-b)(b-c)(a-c)}=1$
$\Leftrightarrow (x-c)[(a^2-b^2)-x(a-b)-c(a-b)]=(a-b)(b-c)(a-c)$
$\Leftrightarrow (x-c)(a-b)(a+b-x-c)=(a-b)(b-c)(a-c)$
$\Leftrightarrow (x-c)(a+b-x-c)-(b-c)(a-c)=0$
$\Leftrightarrow (x-a)(x-b)=0$
Vậy $S =$ {$a;b$}



#358824 Tìm GTNN của: $A=x^2+6y^2+14z^2-8yz+6zx+4xy$

Đã gửi bởi yellow on 04-10-2012 - 17:16 trong Bất đẳng thức và cực trị

Bài này không có điều kiện sao ? Vậy cho $x \to +\infty$ thì $A \to +\infty$ nên không có giá trị nhỏ nhất :)

Bài này theo cách làm của nó thì phải đưa được về bình phương cộng hằng số, nhưng mình nhóm mãi mà không được, Nhân vào rồi nhóm cũng không xong. Mà thầy mình bảo là không sai đề. hix hix... :( :(



#379149 Chứng minh trọng tâm $G$ của $\Delta ABC$ cố định

Đã gửi bởi yellow on 20-12-2012 - 21:20 trong Hình học


$BC \cap (O;r) = D$
Hạ $OI \perp MD$. Dễ dàng chứng minh: $A,O,D:thẳng hàng$
Không mất tính tổng quát, giả sử B,M,I,D,C thẳng hàng theo thứ tự đó
Lần lượt theo Pythagore, ta có:
$MA^2+MB^2+MC^2=4OI^2+(IB-IM)^2+(MI+IC)^2$
$=4OI^2)+IB^2-2BI.IM+IM^2+IM^2+2IM.IC+IC^2$
$=2(r^2+R^2):const$
b, Gọi $G$ là trọng tâm $\triangle ABC$, mà $AI$ là trung tuyến.
Vậy với cách xác định điểm $G$ như trên, $G$ cũng là trọng tâm $\triangle AMD$
$\Rightarrow \frac{MG}{MO} = \frac{2}{3}$
Mà $O:const \Rightarrow G:const$

Làm thế nào để c/m $A, O, D$ thằng hàng anh. Em chứng minh mà chẳng biết đúng sai thế nào nữa!



#358743 Tìm GTNN của: $A=x^2+6y^2+14z^2-8yz+6zx+4xy$

Đã gửi bởi yellow on 04-10-2012 - 11:36 trong Bất đẳng thức và cực trị

Tìm GTNN của: $A=x^2+6y^2+14z^2-8yz+6zx+4xy$



#365007 Tìm giá trị lớn nhất của $f(n)$ với $1\leq n\leq 199...

Đã gửi bởi yellow on 26-10-2012 - 19:53 trong Đại số

$f(x)=1$ là sao hả bạn?

minh nhầm, mình sửa lại đề rồi đó bạn



#364954 Tìm giá trị lớn nhất của $f(n)$ với $1\leq n\leq 199...

Đã gửi bởi yellow on 26-10-2012 - 17:04 trong Đại số

Cho $f:\mathbb{N}\rightarrow \mathbb{N}$ và $f(1)=1$;$f(2n)=f(n)$;$f(2n+1)=f(n)+1$. Tìm giá trị lớn nhất của $f(n)$ với $1\leq n\leq 1994$



#365241 Tìm giá trị lớn nhất của $f(n)$ với $1\leq n\leq 199...

Đã gửi bởi yellow on 27-10-2012 - 15:45 trong Đại số

Hướng dẫn:
Dễ thấy, nếu tồn tại hàm số $f$ thỏa đề thì chỉ có một và chỉ một hàm $f$ thỏa đề. Ta chỉ cần chỉ ra sự tồn tại của hàm số $f$.
Đầu tiên, ta có nhận xét: Mọi số tự nhiên $x$ trong hệ thập phân, đều chỉ có duy nhất một cách biểu diễn dưới dạng nhị phân.
Hàm số $f$ xác định như sau: $f(n)$ là số chữ số $1$ trong cách viết hệ nhị phân của $n$. (1)
Hãy chứng minh (1) bằng quy nạp. Với chú ý: Nếu
\[
n = \overline {a_1 a_2 ...a_k } _{\left( 2 \right)} \Rightarrow \left\{ \begin{array}{l}
2n = \overline {a_1 a_2 ...a_k 0} _{\left( 2 \right)} \\
2n + 1 = \overline {a_1 a_2 ...a_k 1} _{\left( 2 \right)} \\
\end{array} \right.
\]
Từ đó, do $1994 = \overline {11111001010} _{\left( 2 \right)}$ để $f(n)$ có GTLN với $1 \le n \le 1994$ thì \[
n = \overline {1111111111} _{\left( 2 \right)} = 1023
\]

Xin hỏi còn có cách nào khác nữa không? Mấy cái vấn đề bạn nếu trong này mình đều chưa học!!



#365735 Tìm giá trị lớn nhất của $f(n)$ với $1\leq n\leq 199...

Đã gửi bởi yellow on 29-10-2012 - 17:15 trong Đại số

Hướng dẫn:
Dễ thấy, nếu tồn tại hàm số $f$ thỏa đề thì chỉ có một và chỉ một hàm $f$ thỏa đề. Ta chỉ cần chỉ ra sự tồn tại của hàm số $f$.
Đầu tiên, ta có nhận xét: Mọi số tự nhiên $x$ trong hệ thập phân, đều chỉ có duy nhất một cách biểu diễn dưới dạng nhị phân.
Hàm số $f$ xác định như sau: $f(n)$ là số chữ số $1$ trong cách viết hệ nhị phân của $n$. (1)
Hãy chứng minh (1) bằng quy nạp. Với chú ý: Nếu
\[
n = \overline {a_1 a_2 ...a_k } _{\left( 2 \right)} \Rightarrow \left\{ \begin{array}{l}
2n = \overline {a_1 a_2 ...a_k 0} _{\left( 2 \right)} \\
2n + 1 = \overline {a_1 a_2 ...a_k 1} _{\left( 2 \right)} \\
\end{array} \right.
\]
Từ đó, do $1994 = \overline {11111001010} _{\left( 2 \right)}$ để $f(n)$ có GTLN với $1 \le n \le 1994$ thì \[
n = \overline {1111111111} _{\left( 2 \right)} = 1023
\]

Bạn ơi, bạn có thể chứng minh ($1$) bằng cách quy nạp dùm mình được không? Mình làm mãi mà vẫn không chứng minh được.



#378658 Tìm các giá trị nguyên $x, y$ thoả mãn đẳng thức: $(y+2)x^2+1=...

Đã gửi bởi yellow on 18-12-2012 - 20:30 trong Phương trình, hệ phương trình và bất phương trình

Ta có $x^{2}-5x+14=x^{2}-6x+9+x+5=(x-3)^{2}+x+5\geq x+5\geq x+1+4\geq 4\sqrt{x+1}$
Dấu "=" xãy ra khi x=3

Bạn ơi cho mình hỏi vì sao $x+1+4\geq 4\sqrt{x+1}$



#378622 Tìm các giá trị nguyên $x, y$ thoả mãn đẳng thức: $(y+2)x^2+1=...

Đã gửi bởi yellow on 18-12-2012 - 18:13 trong Phương trình, hệ phương trình và bất phương trình

a) Giải phương trình: $4\sqrt{x+1}=x^2-5x+14$
b) Tìm các giá trị nguyên $x, y$ thoả mãn đẳng thức: $(y+2)x^2+1=y^2$



#379113 Tính độ dài $AB$

Đã gửi bởi yellow on 20-12-2012 - 19:05 trong Hình học

Cho $2$ đường tròn $(O;R)$ và $(O';R')$ tiếp xúc ngoài nhau ($R>r$). $AB$ là tiếp tuyến chung của hai đường tròn
($A\in (O;R),B\in (O';R')$). Tính độ dài $AB$



#363272 Tích của $2$ số lẻ liên tiếp, $2$ số chẵn liên tiếp có ph...

Đã gửi bởi yellow on 20-10-2012 - 18:00 trong Số học

Tích của $2$ số lẻ liên tiếp, $2$ số chẵn liên tiếp có phải là số chính phương không? Chứng minh.



#379142 Tính độ dài $AB$

Đã gửi bởi yellow on 20-12-2012 - 21:03 trong Hình học

Anh ơi, làm thế nào c/m được hai tam giác $AMK$ và $BO'K$ đồng dạng vậy anh



#358243 Giải và biện luận phương trình $\frac{x-a}{x+1}=\frac{x+3}{x-1...

Đã gửi bởi yellow on 02-10-2012 - 12:00 trong Hàm số - Đạo hàm

Giải và biện luận phương trình

a) $\frac{x-a}{x+1}=\frac{x+3}{x-1}$

ĐKXĐ: $x\neq \pm 1$
Ta có phương trình trên tương đương với:
$(x-a)(x-1)=(x+3)(x+1)<=>x^2-x-ax+a=x^2+4x+3$
$<=>x(a+5)=-a-3$
Nếu $a = -5$ thì $0x=-8$ phương trình vô nghiệm
Nếu $a\neq -5$ thì $x=\frac{-a-3}{a+5}$
$x=\frac{-a-3}{a+5}$ là nghiệm của phương trình đã cho $<=> \frac{-a-3}{a+5}\neq 1$ và $\frac{-a-3}{a+5}\neq -1$ $<=> a\neq -4$
Đến đây, kết luận nữa là xong, các câu còn lại tương tự mà làm.



#379135 Tính độ dài $AB$

Đã gửi bởi yellow on 20-12-2012 - 20:21 trong Hình học

2 đường tròn ngoài nhau thì không có công thức tổng quát dựa trên bk 2 đường tròn đâu em

Anh ơi, em sửa lại đề rồi, anh xem giúp em với!



#359686 khẩn cấp

Đã gửi bởi yellow on 07-10-2012 - 10:09 trong Số học

ai giúp mình giải bài này với.chứng minh ($2^{2^{m}}$;$2^{2^{n}}$) với m$\neq$n

Bạn kiểm tra lại đề được không?



#379132 Tính độ dài $AB$

Đã gửi bởi yellow on 20-12-2012 - 20:10 trong Hình học

Độ dài tiếp tuyến chung sẽ phụ thuộc vào 2 bán kính, và khoảng cách giữa 2 tâm đường tròn nữa đó em

Tiếp tuyến chung trong và tiếp tuyến chung ngoài đều không được hả anh?



#363312 Tích của $2$ số lẻ liên tiếp, $2$ số chẵn liên tiếp có ph...

Đã gửi bởi yellow on 20-10-2012 - 19:36 trong Số học

Giải như sau:
Gọi hai số lẻ (hai số chẵn) là $a,a+2$
Suy ra $a(a+2)=t^2 \Rightarrow a^2+2a=t^2 \Rightarrow a^2+2a+1=t^2+1 \Rightarrow (a+1)^2=t^2+1$ đến đây quá đơn giản

Đến đây thì nói luôn là ko tồn tại luôn hay là phải cm tiếp bạn



#381516 Tính $M=(x^{4}-\frac{1}{x^{4}...

Đã gửi bởi yellow on 29-12-2012 - 11:10 trong Đại số

Xem lại đề bạn nhé. $a^{3}+b^{3}+c^{3}=abc$ mà.

Bạn xem lại đề bài đi, bài này không thể làm được với điều kiện đó đâu



#363415 Tìm $n\in \mathbb{N}$ để các số sau là số chính...

Đã gửi bởi yellow on 20-10-2012 - 22:24 trong Số học

mình xin giải bài a
đặt $9+2^{n}=a^{2}$
=>$2^{n}=(a-3)(a+3)$
=>$a-3=2^{x} ,a+3=2^{y}$ với x+y=n
$=>2^{y}-2^{x}=6$
$=>2^{x}(2^{y-x}-1)=6$
tới đây dễ rồi.
cây b cũng làm tương tự

Bạn có thể làm rõ hơn được không, đến cuối mình vẫn không tìm ra được đáp số

Mình cảnh cáo bạn yellow về spam quá nhiều-nguyenta98



#357763 $\left\{\begin{matrix} x_{1}x_{2}x_{3}=x_{1}+x_{2}+x_{3}...

Đã gửi bởi yellow on 30-09-2012 - 12:27 trong Phương trình, hệ phương trình và bất phương trình

Giải hệ phương trình:
$\left\{\begin{matrix} x_{1}x_{2}x_{3}=x_{1}+x_{2}+x_{3}\\ x_{2}x_{3}x_{4}=x_{2}+x_{3}+x_{4}\\ ..........\\ ..........\\ x_{1985}x_{1986}x_{1987}=x_{1985}+x_{1986}+x_{1987}\\ x_{1986}x_{1987}x_{1}=x_{1986}+x_{1987}+x_{1}\\ x_{1987}x_{1}x_{2}=x_{1987}+x_{1}+x_{2}\\ \end{matrix}\right.$



#359578 Bất đẳng thức Chebyshev

Đã gửi bởi yellow on 06-10-2012 - 22:03 trong Bất đẳng thức và cực trị

Bài 3: Cho a, b, c >0 thỏa: $\frac{1}{1+a+b}+\frac{1}{1+b+c}+\frac{1}{1+c+a}\geq 1$
CMR: a+b+c $\geq$ ab+bc+ca

Sử dụng bất đẳng thức $Cauchy-Schwarz$, ta có:
$\frac{1}{a+b+1}=\frac{a+b+c^2}{(a+b+1)(a+b+c^2)}\leq \frac{a+b+c^2}{(a+b+c)^2}$
Tương tự ta có: $\frac{1}{b+c+1}\leq \frac{b+c+a^2}{(a+b+c)^2}$
$\frac{1}{c+a+1}\leq \frac{c+a+b^2}{(a+b+c)^2}$
$\Rightarrow \frac{1}{1+a+b}+\frac{1}{1+b+c}+\frac{1}{1+c+a}\frac{a^2+b^2+c^2+2(a+b+c)}{(a+b+c)^2}$
Do $1\leq \frac{1}{1+a+b}+\frac{1}{1+b+c}+\frac{1}{1+c+a}$ nên ta có:
$a^2+b^2+c^2+2(a+b+c)\geq (a+b+c)^2\Rightarrow a+b+c\geq ab+bc+ca$
Bất đẳng thức được chứng minh. Đẳng thức xảy ra $\Leftrightarrow a=b=c=1$



#359580 Bất đẳng thức Chebyshev

Đã gửi bởi yellow on 06-10-2012 - 22:06 trong Bất đẳng thức và cực trị

Xin lỗi bạn vì mình đã không sử dụng $Chebyshev$, nếu bạn cần cách chứng minh bằng $Chebyshev$ thì để mình xem lại đã, coi như hai bài trên là cách khác đi. :wub: :wub: :wub:



#363413 Tìm $n\in \mathbb{N}$ để các số sau là số chính...

Đã gửi bởi yellow on 20-10-2012 - 22:20 trong Số học

c, $A=n^4+2n^3+2n^2+n+7$.
Ta có $(n^2+n)^2<A<(n^2+n+3)^2$
$\implies \left [ \begin{array}{l} A=(n^2+n+1)^2 \\ A=(n^2+n+2)^2 \end{array} \right.$

TH1: Nếu $A=(n^2+n+1)^2 \implies n^4+2n^3+2n^2+n+7=n^4+2n^3+3n^2+2n+1$
$\implies n^2+n=6 \implies n(n+1)=6$.
Do $n \in \mathbb{N} \implies n=2$.

TH2: Nếu $A=(n^2+n+2)^2 \implies n^4+2n^3+2n^2+2n+7=n^4+2n^3+5n^2+4n+4$
$\iff 3n^2+2n=3 \iff n(3n+2)=3$. Hiển nhiên TH này không tìm được $n$ thỏa mãn

$\boxed{ \text{Kết luận.} }.$ Vậy $\boxed{n=2}$.

Sao chứng minh được cái này bạn: $(n^2+n)^2<A<(n^2+n+3)^2$